1 / 24

Modular Arithmetic

Modular Arithmetic. Lecture 9: Oct 5. Modular Arithmetic. Def : a  b (mod n) iff n|( a - b) iff a mod n = b mod n. e.g. 12  2 (mod 10) 107  207 (mod 10) 7  1 (mod 2) 1  -1 (mod 2) 13  -1 (mod 7) -15  0 (mod 5). Modular Addition.

milos
Download Presentation

Modular Arithmetic

An Image/Link below is provided (as is) to download presentation Download Policy: Content on the Website is provided to you AS IS for your information and personal use and may not be sold / licensed / shared on other websites without getting consent from its author. Content is provided to you AS IS for your information and personal use only. Download presentation by click this link. While downloading, if for some reason you are not able to download a presentation, the publisher may have deleted the file from their server. During download, if you can't get a presentation, the file might be deleted by the publisher.

E N D

Presentation Transcript


  1. Modular Arithmetic Lecture 9: Oct 5

  2. Modular Arithmetic Def:a  b (mod n) iff n|(a - b) iff a mod n = b mod n. e.g. 12  2 (mod 10) 107  207 (mod 10) 7  1 (mod 2) 1  -1 (mod 2) 13  -1 (mod 7) -15  0 (mod 5)

  3. Modular Addition Lemma: If a  c (mod n), and b  d (mod n) then a+b  c+d (mod n). Example 1 12  2 (mod 10), 25  5 (mod 10) => 12 + 25 (mod 10)  2 + 5 (mod 10)  7 (mod 10) Example 2 87  2 (mod 17), 222  1 (mod 17) => 87 + 222 (mod 17)  2 + 1 (mod 17)  3 (mod 17) Example 3 101  2 (mod 11), 141  -2 (mod 11) => 101 + 141 (mod 11)  0 (mod 11)

  4. Modular Addition Lemma: If a  c (mod n), and b  d (mod n) then a+b  c+d (mod n). a  c (mod n) => a = c + nx for some integer x b  d (mod n) => b = d + ny for some integer y To show a+b  c+d (mod n), it is equivalent to showing that n | (a+b-c-d). Consider a+b-c-d. a+b-c-d = (c+nx) + (d+ny) – c –d = nx + ny. It is clear that n | nx + ny. Therefore, n | a+b-c-d. We conclude that a+b  c+d (mod n). Proof

  5. Modular Multiplication Lemma: If a  c (mod n), and b  d (mod n) then ab  cd (mod n). Example 1 9876  6 (mod 10), 17642  2 (mod 10) => 9876 * 17642 (mod 10)  6 * 2 (mod 10)  2 (mod 10) Example 2 10987  1 (mod 2), 28663  1 (mod 2) => 10987 * 28663 (mod 2)  1 (mod 2) Example 3 999  5 (mod 7), 674  2 (mod 7) => 999 * 674 (mod 7)  5 * 2 (mod 7)  3 (mod 7)

  6. Modular Multiplication Lemma: If a  c (mod n), and b  d (mod n) then ab  cd (mod n). Proof a  c (mod n) => a = c + nx for some integer x b  d (mod n) => b = d + ny for some integer y To show ab  cd (mod n), it is equivalent to showing that n | (ab-cd). Consider ab-cd. ab-cd = (c+nx) (d+ny) – cd = cd + dnx + cny + n2xy – cd = n(dx + cy + nxy). It is clear that n | n(dx + cy + nxy). Therefore, n | ab-cd. We conclude that ab  cd (mod n).

  7. Exercise 1444 mod 713 = 144 * 144 * 144 * 144 mod 713 = 20736 * 144 * 144 mod 713 = 59 * 144 * 144 mod 713 = 8496 * 144 mod 713 = 653 * 144 mod 713 = 94032 mod 713 = 629 mod 713 20736 * 20736 mod 713 = 59 * 59 mod 713 = 3481 mod 713 = 629 mod 713

  8. Application Is a number written in decimal evenly divisible by 9 if and only if the sum of its digits is a multiple of 9? Example 1. 9333234513171 is divisible by 9. 9+3+3+3+2+3+4+5+1+3+1+7+1 = 45 is divisible by 9. Example 2. 128573649683 is not divisible by 9. 1+2+8+5+7+3+6+4+9+6+8+3 = 62 is not divisible by 9. NO A coincidence? This can be proved easily using modular arithmetic.

  9. Application Claim. A number written in decimal is divisible by 9 if and only if the sum of its digits is a multiple of 9? Hint: 10  1 (mod 9). Let the decimal representation of n be dkdk-1dk-2…d1d0. This means that n = dk10k + dk-110k-1 + … + d110 + d0 Note that di10i mod 9 = (di mod 9) (10i mod 9) mod 9 = (di mod 9) (10 mod 9) (10 mod 9) … (10 mod 9) mod 9 = (di mod 9) (1 mod 9) (1 mod 9) … (1 mod 9) mod 9 = di mod 9 i terms

  10. Application Claim. A number written in decimal is divisible by 9 if and only if the sum of its digits is a multiple of 9? Hint: 10  1 (mod 9). Let the decimal representation of n be dkdk-1dk-2…d1d0. This means that n = dk10k + dk-110k-1 + … + d110 + d0 Note that di10i mod 9 = di mod 9. Hence n mod 9 = (dk10k + dk-110k-1 + … + d110 + d0) mod 9 = (dk10k mod 9 + dk-110k-1 mod 9 + … + d110 mod 9 + d0 mod 9) mod 9 = (dk mod 9 + dk-1 mod 9 + … + d1 mod 9 + d0 mod 9) mod 9 = (dk + dk-1 + … + d1 + d0) mod 9

  11. Multiplication Inverse The multiplicative inverse of a number a is another number a’ such that: a · a’ 1 (mod n) For real numbers, every nonzero number has a multiplicative inverse. For integers, only 1 has a multiplicative inverse. An interesting property of modular arithmetic is that there are multiplicative inverse for intgers. For example, 2 * 5 = 1 mod 3, so 5 is a multiplicative inverse for 2 under modulo 3 (and vice versa). Does every number has a multiplicative inverse in modular arithmetic?

  12. Multiplication Inverse Does every number has a multiplicative inverse in modular arithmetic?

  13. Multiplication Inverse What is the pattern?

  14. Multiplication Inverse Why 2 does not have a multiplicative inverse under modulo 6? Suppose it has a multiplicative inverse y. 2y  1 (mod 6) => 2y = 1 + 6x for some integer x => y = ½ + 3x This is a contradiction since both x and y are integers. Claim. An integer k does not have an multiplicative inverse under modulo n, if k and n have a common factor >= 2 (gcd(k,n) >= 2). Proof. Same as above. Leave it as an exercise.

  15. Multiplication Inverse What about if gcd(k,n)=1? Would k always have an multiplicative inverse under modulo n? Theorem. If gcd(k,n)=1, then have k’ k·k’  1 (mod n). k’ is an inversemod n of k gcd(k,n)=spc(k,n) Proof: Since gcd(k,n)=1, there exist s and t so that sk + tn = 1. So tn = 1 - sk This means n | 1 – sk. This means that 1 – sk  0 (mod n). This means that 1 = sk (mod n). So k’ = s is an multiplicative inverse for k.

  16. Cancellation Note that  (mod n) a lot like =. If a b (mod n), then a+c b+c (mod n). If a b (mod n), then ac bc (mod n) However, if ac bc (mod n), it is not necessarily true that a b (mod n). For example, 4·2  1·2 (mod 6), but 4  1 (mod 6) There is no general cancellation in modular arithmetic.

  17. Cancellation Why a·k  b·k (mod n) when a ≠ b? Without loss of generality, assume 0 < a < n and 0 < b < n. Because if a·k  b·k (mod n), then also (a mod n)·k  (b mod n)·k (mod n). smaller than n. This means that ak = bk + nx. This means that (a-b)k = nx, which means a-b=(nx)/k. Since 0 < a < n and 0 < b < n, it implies that –n < a-b < n. Therefore, nx/k must be < n. For this to happen, n and k must have a common divisor >= 2! Okay, so, can we say something when gcd(n,k)=1?

  18. Cancellation Claim: If i·k  j·k (mod n), andgcd(k,n) = 1, then i  j (mod n) For example, multiplicative inverse always exists if n is a prime! Proof. Since gcd(k,n) = 1, there exists k’ such that kk’  1 (mod n). i·k  j·k (mod n). => i·k·k’  j·k·k’ (mod n). => i  j (mod n) This makes arithmetic modulo prime a field, a structure that “behaves like” real numbers. Arithmetic modulo prime is very useful in coding theory.

  19. Fermat’s Little Theorem If p is prime & k not a multiple of p, then we can cancel k. So k mod p, 2k mod p, …, (p-1)k mod p are all different. This means that k mod p, 2k mod p,…,(p-1)k mod p must be a permutation of 1, 2, ···, (p-1) (each number appears exactly once)

  20. Fermat’s Little Theorem Theorem: If p is prime & k not a multiple of p 1  kp-1 (mod p) Proof. 1·2···(p-1)  (k mod p · 2k mod p··· (p-1)k mod p) mod p  (k·2k ··· (p-1)k) mod p  (kp-1)·1·2 ··· (p-1) (mod p) So, by cancelling 1·2 ··· (p-1) on both sides, we have 1  kp-1 (mod p) A permutation

  21. Wilson’s Theorem Theorem:p is a prime if and only if (p-1)!  -1(mod p) First we consider the easy direction. If p is not a prime, assume p >= 5, (for p=4, 3!  2 (mod 4) ) Then p=qr for some 2 <= q < p and 2 <= r < p. If q ≠ r, then both q and r appear in (p-1)!, and so (p-1)! 0 (mod p). If q = r, then p = q2 > 2q (since we assume p > 5 and thus q > 2). then both q and 2q are in (p-1)!, and so again (p-1)!  0 (mod p).

  22. Wilson’s Theorem Theorem:p is a prime if and only if (p-1)!  -1(mod p) To prove the more interesting direction, first we need a lemma. Lemma. If p is a prime number, x2 1 (mod p) if and only if x  1 (mod p) or x  -1 (mod p) Proof. x2 1 (mod p) iff p | x2- 1 iff p | (x– 1)(x + 1) iff p | (x – 1) or p | (x+1) iff x  1 (mod p) or x  -1 (mod p) Lemma:p prime and p|a·b iffp|a or p|b.

  23. Wilson’s Theorem Theorem:p is a prime if and only if (p-1)!  -1(mod p) Let’s get the proof idea by considering a concrete example. 10! 1·2·3·4·5·6·7·8·9·10 mod 11  1·10·(2·6)·(3·4)·(5·9)·(7·8) mod 11  1·-1·(1)·(1)·(1)·(1) mod 11  -1 mod 11 Besides 1 and 10, the remaining numbers are paired up into multiplicative inverse!

  24. Wilson’s Theorem Theorem:p is a prime if and only if (p-1)!  -1(mod p) Proof. Since p is a prime, every number from 1 to p-1 has a multiplicative inverse. By the Lemma, every number 2 <= k <= p-2 has an inverse k’ with k≠k’. Since p is odd, the numbers from 2 to p-2 can be grouped into pairs (a1,b1),(a2,b2),…,(a(p-3)/2,b(p-3)/2) so that aibi 1 (mod p) Therefore, (p-1)!  1·(p-1)·2·3·····(p-3)·(p-2) (mod p) 1·(p-1)·(a1b1)·(a2b2)·····(a(p-3)/2b(p-3)/2) (mod p) 1·(-1)·(1)·(1)·····(1) (mod p) -1 (mod p)

More Related